22
$\begingroup$

user$xxxxx$ posted (and then deleted) the following question which I think deserves to be here:

Prove that the tensor product of two Artinian modules is Artinian.

  • 1
    The OP didn't mention if the ring is commutative or not, so I suggest to consider both cases.2012-12-25
  • 4
    The following paper by George Bergman discusses the case where the base ring is commutative: http://arxiv.org/abs/1108.2520v12012-12-26
  • 0
    @Rankeya I was aware of this paper (google is a good friend!) when I've (re)posted this question, but I expect some other opinions.2012-12-26
  • 7
    I think this question is more suitable for MathOverflow.2012-12-30
  • 2
    This is a very interesting question, as it is so easy to formulate, but seemingly not as easy to answer. Did you by any chance find an answer in the meantime / post this on MO? I'd really like to know more about this :)2013-01-15
  • 1
    @Randal'Thor The answer is YES, but I don't know yet an elementary proof.2013-01-16
  • 0
    Could you add the proof to the question?2013-01-30
  • 0
    You can find one in the paper of Bergman quoted in the second comment above.2013-01-30

1 Answers 1